Oral ENS Ulm, 36 planches - Page 3 — Les-mathematiques.net The most powerful custom community solution in the world

Oral ENS Ulm, 36 planches

135

Réponses

  • Merci LOU16 pour ta solution.

    Pour revenir sur l'exercice 1, je cherche une preuve sans utiliser les chaînes de Markov.
    Notant $\forall n \geq 1 \, x_n$ le nombre de configurations gagnantes pour le tour $3n$ (i.e. le nombre d'anagrammes à $3n$ lettres comportant $n$ fois la lettre F et $2n$ fois la lettre P pour lesquels on obtient pour la première fois deux fois plus de P que de F au tour $3n$), a-t-on bien $x_n = {3n \choose n} - 3x_{n - 1}$?

    Je compte, pour le tour $3n$, ${3n \choose n}$ anagrammes à $3n$ lettres comportant $n$ fois la lettre F et $2n$ fois la lettre P auquel il faut retirer le nombre de configurations gagnantes du tour d'avant multiplié par le nombre de façons de placer les deux nouvelles lettres P et la nouvelle lettre F.

    Une simulation me montre que la formule ne tient pas je ne vois pas où est l'erreur dans mon raisonnement.
  • @side Je pense que tu confonds... On s'intéresse à $g$ qui est la limite simple des dérivées $n$-ième de $f.$
    Je pense que l'énoncé est hyper difficile (voire faux, je ne sais pas...) sans ajouter une condition de régularité sur la limite $g$ (disons localement bornée suffirait).
  • Oui, c'est mieux et plus simple! (en utilisant le formalisme des limites inf et sup, on peut simplifier un peu...)
  • sevaus
    la bonne formule est $x_n = \displaystyle{3n \choose n} - \sum_{k=1}^{n-1} {3(n-k) \choose n-k}x_k$.
  • Planche 16 (premier exercice)

    *On définit pour $x\in[0,1],$ $p_{0}(x)=1;p_{1}(x)=x$ et pour tout $n\geq 2,$ $\displaystyle p_{n}(x)=\int_{0}^{x}dx_{1}\int_{0}^{1-x_{1}}dx_{2}\ldots\int_{0}^{1-x_{n-1}}dx_{n}.$
    On a par le caractère muet des variables d'intégration, pour tout $x\in[0,1]$ et pour tout $n\geq 0$ : $\displaystyle p'_{n+1}(x)=p_{n}(1-x).$

    Soit $y$ vérifiant $0<y<1$ (mais on verra à la fin que $\vert y\vert <\frac{\pi}{2}$ convient).
    Pour $x\in[0,1],$ considérons $\displaystyle \phi_{y}(x)=\sum_{n\geq 0}p_{n}(x)y^{n}.$

    En dérivant termes à termes (car la convergence de la série dérivée en $x$ est normal sur $[0,1]$ à $y$ fixé dans le régime prescrit),
    on obtient pour tout $x\in[0,1],$ $\displaystyle \phi'_{y}(x)=\sum_{n\geq 0}p'_{n}(x)y^{n}=y\phi_{y}(1-x).$
    On a alors pour tout $x\in[0,1],$ $\phi''_{y}(x)=-y^{2}\phi_{y}(x).$

    **Ainsi, il existe $A,B\in\mathbb{R}$ tels que pour tout $x\in[0,1],$ $\phi_{y}(x)=A\cos(xy)+B\sin(xy).$
    Comme $\phi_{y}(0)=1$ et $\phi'_{y}(1)=y,$ on obtient : $A=1$ et $\displaystyle B=\frac{1+\sin(y)}{\cos(y)}.$

    *** La quantité recherchée dans l'énoncé est alors $\displaystyle \phi_{y}(1)=\frac{1+\sin(y)}{\cos(y)}.$
    En particulier pour $y=\frac{\pi}{6},$ il vient que la quantité voulue est $\displaystyle S:=\phi_{\frac{\pi}{6}}(1)=\sqrt{3}.$
  • +Bonjour,

    $\underline{\text{Planche 1}}.$
    @Sevaus.

    L' origine de ton "erreur" me donne une idée de solution plus simple que celle contenue dans le troisième lien indiqué par Jandri. (ce message)
    Je me permets de désigner par $a_n$ ce que tu notes $x_n:$
    $$ a_n = \text {Card} \left\{ (e_i)_{1\leqslant i \leqslant 3n} \in\{0,1\}^{3n} \mid \displaystyle \sum _{i=1}^{3n} e_i = n,\:\: \forall k \in [\![1;n-1]\!], \sum _{k=1}^{3k} e_i \neq k \right \}$$.
    La "bonne" relation de récurrence entre les $a_n$ est: $a_0=0, \quad\: \forall n \in \N^*,\:\displaystyle \binom {3n}{ n} = \sum_{k=0} ^n \binom {3k} k a_{n-k}.\:\:$
    Ainsi si l'on note: $\forall x\in [0; \frac 4{27}[,\:\:S(x) =\displaystyle \sum_{n=0}^{+\infty} \binom {3n}n x^n, \quad A(x) = \sum _{n=0}^{+\infty} a_nx^n, \quad\quad\:$ on a: $\quad S(x) -1 = A(x) S(x) . \quad \quad(\star)$
    $\boxed {\text{La probabilité que le jeu s'arrête est }\:A(\frac 18).}\quad$ Il reste donc à calculer $S(\frac18).$
    Il est alors bon de remarquer que: $\:\displaystyle \forall n \in \N,\:\binom {3n} n \dfrac1{8^n}=\dfrac 1{2\mathrm i \pi} \int _{\gamma} \left( \dfrac{(1+z)^3} {8z}\right) ^n \dfrac {\mathrm d z}z\quad$ où $\: \gamma :\left \{ \begin{array} {ccc} [0;2\pi] &\longrightarrow & \C \\ t & \longmapsto & \dfrac{ \mathrm e^{\mathrm i t}}2 \end{array} \right. $
    $\forall z \in \gamma, \:\: \left| \dfrac {(1+z)^3}{8z} \right |<1.\:$ En notant $f(z) = z^{-1} \left( 1- \dfrac {(1+z)^3}{8z} \right) ^{-1} = \dfrac 8{ (1-z)(z^2 +4z-1)}$, un peu d'analyse complexe conduit à:
    $ S\Big(\dfrac18 \Big) = \dfrac 1{2 \mathrm i \pi}\displaystyle \int_{\gamma} f(z)\: \mathrm dz=\text {Res} ( f , \sqrt 5 -2) = \dfrac {3+\sqrt 5}{\sqrt 5 }.\quad $ Puis à l'aide de $(\star),\:$ on parvient à: $\quad \boxed { A \Big( \dfrac 18 \Big) = \dfrac 3{3+\sqrt 5} .}$
  • Planche 19 exo 2?
    Le 😄 Farceur


  • ok pour planche 16 exercice1
  • @etanche
    C'est pour repondre à la question. BJ considère $\displaystyle \phi_{y}(x)=\sum_{n\geq 0}p_{n}(x)y^{n}.$ et la question c'est exactement le calcul de $ \phi_{\frac {\pi}2}(1)$
    edit tippo à lire $\frac {\pi}6$
    Le 😄 Farceur


  • @gebrane : ben non justement... c'est bien $\displaystyle \phi_{\frac{\pi}{6}}(1)$ qui est demandé...
  • Planche 19 (Exercice 2)
    i) Le premier cas est non.
    Supposons par l'absurde qu'il existe $f : \mathbb{R} \rightarrow \mathbb{R}$ $\mathcal{C}^{1}$ telle que $\forall x\in\mathbb{R},\mbox{ } f\circ f'(x)=x.$

    *On voit que $f'$ est injective sur $\mathbb{R}$. Etant continue, $f'$ est strictement monotone sur $\mathbb{R}.$
    On suppose que $f'$ est strictement croissante (le cas $f'$ strictement décroissante se traite de la même manière).

    **Notons $\displaystyle l=\lim_{x\rightarrow +\infty}f'(x).$ Supposons que $l<+\infty.$
    On a alors par continuité de $f$ et par unicité de la limite : $\displaystyle f(l)=+\infty,$ une contradiction!
    Ainsi, $l=+\infty.$
    (On procède de même pour montrer que $\displaystyle \lim_{x\rightarrow -\infty}f'(x)=-\infty$).
    Ainsi, $f'$ réalise une bijection de $\mathbb{R}$ sur $\mathbb{R}.$

    ***Montrons alors $f$ est injective sur $\mathbb{R}.$
    Soit $x,y\in\mathbb{R}$ tels que $f(x)=f(y).$ On a alors $a,b\in\mathbb{R}$ tels que $x=f'(a);y=f'(b).$
    On obtient donc $a=f\circ f'(a)=f(x)=f(y)=f\circ f'(b)=b.$ Et ainsi, $x=y.$
    Ainsi, $f$ étant continue sur $\mathbb{R},$ $f$ est strictement monotone sur $\mathbb{R}$ également.
    On suppose alors $f$ strictement croissante (le cas $f$ strictement décroissante se traite de la même manière).

    ****Ainsi, pout tout $x\in\mathbb{R},$ $\displaystyle f'(x)\geq 0,$ contredisant le fait que $f'$ est surjective...

    ii) Le deuxième cas est oui.
    Supposons qu'il existe $f : \mathbb{R}^{+*} \rightarrow \mathbb{R}^{+*}$ $\mathcal{C}^{1}$ telle que $\forall x\in\mathbb{R}^{+},\mbox{ } f\circ f'(x)=x.$

    -Nécessairement, $f'$ est à valeurs dans $\mathbb{R}^{+*}.$ Ainsi, $f$ est strictement croissante sur $\mathbb{R}^{+*}.$
    Comme $f'$ est strictement monotone sur $\mathbb{R}^{+*}$ (le point * précédent vaut toujours pour cette partie), alors $f'$ est strictement croissante (compte-tenu de la relation $f\circ f'=\mbox{Id}_{\mathbb{R}^{+*}}$).

    -On a comme précédemment : $\displaystyle \lim_{x\rightarrow +\infty}f'(x)=+\infty.$

    Supposons que $\displaystyle \lim_{x\rightarrow 0^{+}}f'(x)=l>0.$
    On a alors par continuité de $f,$ $f(l)=0,$ une contradiction!

    Ainsi, $f'$ réalise une bijection de $\mathbb{R}^{+*}$ dans $\mathbb{R}^{+*}.$

    -Montrons alors que $f^{-1}=f'.$

    Soit $x>0.$ On a $a(x)>0$ tel que $f'(a(x))=x$ mais alors en composant par $f$, il vient $f(x)=a(x)$ i.e. $f'\circ f(x)=x.$

    *Voilà ce que l'on peut dire (sans intuiter la forme de la solution...) mais @Side l'a fait un peu plus loin... On peut chercher $f :x \mapsto ax^{b}$ sur $\mathbb{R}^{+*}$ pour des bonnes constantes $a$ et $b$ (cf détails plus bas!).
  • Une bout de solution de la planche 29 :

    Comme l'indique les éléments de discussion, il faut se ramener à des formes plus simples de $A$ et $B$ : pour des raisons qu'i vont apparaitre clair plus tard je vais prendre $A$ et $B$ à coefficient complexe au lieu de réel on peut commencer par traiter le cas où $A$ et $B$ sont diagonales qui ne pose pas trop de soucis, puis montrer qu'on peut remplacer $A$ et $B$ par des matrices semblables pourvu que la matrice de passage soit la même. Comme $A$ et $B$ commutent on a des résultats de réduction simultané, en particulier comme on a montré le résultat pour $A$ et $B$ diagonale, on peut étendre le résultat à $A$ et $B$ diagonalisable puisque si $A$ et $B$ sont diagonalisable et commutent elles sont diagonale dans la même base. Enfin pour conclure (c'est la que s'être placer chez les complexes est utile) on écrit $A$ et $B$ comme la somme d'une matrice diagonalisable et une matrice nilpotente qui sont des polynômes en $A$ (resp. $B$) (Dunford) et on conclut.

    pour la seconde question, je pense que le résultat ne tient pas principalement car l'absence de réduction simultané semble être une obstruction assez insurmontable, peut-être qu'on peut bidouiller à l'aide de ce genre de matrices pour construire un contre exemple :

    $$\begin{pmatrix} 0 & -1 \\
    1+ \epsilon & 0\end{pmatrix}, \begin{pmatrix} 0 & 1 + \epsilon \\
    -1 & -1\end{pmatrix}$$

    car on a $ (\begin{pmatrix} 0 & -1 \\
    1 & 0\end{pmatrix} \begin{pmatrix} 0 & 1 \\
    -1 & -1\end{pmatrix})^k =\begin{pmatrix} 1 & k \\
    0 & 1\end{pmatrix} $ mais je n'y ai pas vraiment rélféchis

    Edit : solution mise en claire a la demande de etanche
  • @ viko comment fait on pour lire ta solution , je ne vois que du blanc
    Pourrais-tu le mettre en mode normal ? merci
  • Merci BJ por la planche 19, c'est tres instructif
    Le 😄 Farceur


  • Comment faire pour lire la solution de viko ? Tout est en blanc
  • Pour lire la solution de viko il faut sélectionner le texte.

    Exercice 8.2 (Idée générale) Considérons une matrice solution $M$ de coefficients $a, b, c, d$, la condition de l'énoncé étant que $a+b+c+d=r$. Le polynôme minimal de $M$ est de degré plus petit que $2$, mais $M$ est annulée par $X^6-1=(X+1)(X-1)(X^2+X+1)(X^2-X+1)$ donc le polynôme minimal que l'on cherche doit être $X^2\pm X+1$, supposons que ce soit $X^2-X+1$, c'est aussi le polynôme caractéristique ce qui impose $a+d=1$ et $ad-bc=1$. Avec tout ça on doit pouvoir concocter des coefficients $a, b, c, d$ qui fonctionnent. Si j'ai vraiment dit n'importe quoi dites le moi :)o

    Question. Est-ce que l'énoncé de l'exercice 14.1 est bon ?

    Edit. A la vue de la réponse de side, cet exercice aurait pu être écrit sans la suite $b_n$...

    3.1 [Pour ceux que l'exercice ne passionne pas, voir une solution ici : http://scipp.ucsc.edu/~haber/ph116A/charpoly_11.pdf]
    17.2. [C'est juste une utilisation de la multilinéarité du déterminant, lorsque $B=I_n$ on voit apparaître le développement du déterminant par rapport à une colonne/ligne].
  • @side peux-tu nous écrire une solution pour planche 27 exercice 1 merci
  • Exercice 3.2 (car Mickaël me l'a demandé gentiment! ^^)

    Soit $y>0.$
    Comme $f$ est dérivable, on a pour $x$ proche de $1$ :
    $\displaystyle f(x)=f(1)+(x-1)f'(1)+o(x-1)=1+(x-1)f'(1)+o(x-1)$ et $\displaystyle f(xy)=f(y)+(x-1)yf'(y)+o(x-1).$
    Ainsi, on obtient : $$f(x)f(y)-f(xy)=(x-1)\left( f'(1)f(y)-yf'(y) \right)+o(x-1)\leq 0.$$
    En divisant par $(x-1)$ (en traitant les cas $x>1$ et $x<1$) et en faisant tendre $x$ vers $1$, on obtient : $$yf'(y)=f'(1)f(y).$$
    Il vient alors pour tout $y>0,$ $\displaystyle f(y)=y^{f'(1)}$ (car $f(1)=1$).
    Cependant, $f$ doit être dérivable sur $\mathbb{R}^{+}$ donc $f'(1)\geq 1.$

    Il est facile de vérifier que les fonctions du type $ f: x\mapsto x^{\lambda}$ où $\lambda\geq 1$ sont dérivables sur $\mathbb{R}^{+}$ et satisfont l'inéquation fonctionnelle.
  • @étanche je t'écris une solution ce soir pour le 27.1 si tu es prêt à attendre jusque là. L'idée est de regarder $F(x):=y(x)\exp(-\int_0^x a(u)du)$, dériver et utiliser l'équation et les hypothèses pour montrer la convergence par des moments de monotonie.
  • Exercice 32.3. La condition est $f(-1), f(0)\in \mathbb{Z}$ c'est un résultat bien connu des taupins : on se ramène au cas où les valeurs au bord sont $0$ puis on modifie les polynômes de Bernstein dans la preuve du théorème de Weiredtrass. Si on veut mimer la preuve par convolution ça doit être possible. A noter que si on avait pris le même problème mais sur un segment ne contenant pas d'entiers alors on aurait pu approcher toute fonction continue.
  • Exercice 34.1. C'est compliqué. Des tentatives qui ont échoué m'ont conduit à mener une recherche Google qui donne un article dans lequel ce resultat est démontré :http://www.numdam.org/item/BSMF_1969__97__129_0/
  • Non le 34.1 est facile en considérant la décomposition en facteurs premiers de $g(n)$ dans le cas impair. On montre que les seuls facteurs premiers possibles sont 3 et 5 avec une condition sur les exposants qui implique $n \leq 9$.
  • Sauf que $n= 15$ est solution !
    Mais je suis preneur de ton raisonnement parce que j'ai essayé de raisonner aussi avec des recompositions en facteurs premiers, sans succès.

    Exercice 19.3. Écrivons $P=AB$ alors on voit que comme le coef constant est premier les coefs constants de $A,B$ sont $1, p_n$. Mais cela n'est pas possible car $P$ n'a que des racines de module strictement plus grand que $1$, en effet, on voit que $1$ n'est pas racine et la monotonie des coefficients permet d'utiliser la borne de montel, voir http://www.les-mathematiques.net/phorum/read.php?4,415195,415286 message de Borde.
  • @Mickaël : ok, je suis allé vite sans rien rédiger et j'ai sans doute écrit des bêtises. Les conditions ne me permettent pas de faire mieux pour l'instant, j'y reviendrai dans quelques jours.
  • @Siméon : pas de souci :) Mon lien est juste un aveu de faiblesse de ma part, je n'y suis pas arrivé et je suis persuadé que tu as en tête un truc plus simple ! Et sinon niveau probabilités tu semblais dire que les exos sont simples (plus simples que les années précédentes car je sais si faire !), edt-ce que tu pourrais donner des indications pour les exos de probas restants ?

    Exercice 24.1 : le quasi caractère qui fonctionne est $\lim_n M(g^n)/n$. On prouve que la limite existe en montrant que la suite est de Cauchy par application de l'inégalité triangulaire et itération de l'hypothèse de bornitude.

    Exercice 5.1. On se place dans le cas non borné avec une suite $f(x_n)$ qui tend vers l'infini. On regarde $f(xyx_n)/f(x_n)$ qui tend vers $f(xy)$ mais aussi $f(x)f(y)$. Reste à calcul et la plus petite constante et pour ça je ne sais pas ...
  • Planche 35 (deuxième exercice)

    -Pour $c<0,$ la fonction $\displaystyle f: x\mapsto e^{x}$ satisfait les inéquations différentielles.
    -Pour $c=0,$ les fonctions $\displaystyle f_{\lambda} :x \mapsto e^{\lambda x}$ où $\lambda>1$ satisfont les inéquations différentielles.
    -Pour $c>0,$ il n'y a pas de fonction $\mathcal{C}^{2}$ qui vérifie les deux inéquations différentielles.
    Par l'absurde, supposons qu'il existe une telle fonction $f$...

    *L'idée est la suivante, on a va voir que les inégalités forcent $f'$ et $f$ à être négatives dans un voisinage de $-\infty.$
    Mais alors $f$ est décroissante dans ce voisinage de $-\infty$ et tend vers $+\infty$ en $-\infty,$ contredisant sa négativité!


    **Par la méthode de la variation de la constante (ou par la formule de Duhamel), on a pour tout $x\in\mathbb{R},$
    $$f'(x)=e^{x}\int_{0}^{x}\left(f''(t)-f'(t)\right)e^{-t}dt+f'(0)e^{x} \mbox{ et } f(x)=e^{x}\int_{0}^{x}\left(f'(t)-f(t)\right)e^{-t}dt+f(0)e^{x}.$$

    Il vient alors par croissance de l'intégrale et pour tout $x\leq 0,$ $$f'(x)\leq ce^{x}\int_{x}^{0}-e^{-t}dt+f'(0)e^{x} = c(e^{x}-1)+f'(0)e^{x} \mbox{ et } f(x)\leq ce^{x}\int_{x}^{0}-e^{-t}dt+f(0)e^{x} = c(e^{x}-1)+f(0)e^{x}.$$
    En particulier, on a $\displaystyle \limsup_{x\rightarrow +\infty} f(x) \leq -c<0.$

    Une autre façon d'écrire ceci est de remarquer que le membre de droite tend vers $-c$ en $-\infty$ et donc $f'$ est négative dans un voisinage de $-\infty.$ Ainsi, $f$ est décroissante dans un voisinage de $-\infty$ et donc admet une limite en $-\infty$ qui est strictement négative en passant à la limite dans la deuxième inégalité.

    Finalement, en intégrant entre $x$ et $0$ la première inégalité (lorsque $x\leq 0$), il vient $$f(0)-f(x)\leq (c+f'(0))(1-e^{x})+cx \mbox{ i.e. } f(x)\geq f(0)-(c+f'(0))(1-e^{x})-cx.$$
    Par comparaison, on obtient $\displaystyle \lim_{x\rightarrow -\infty} f(x)=+\infty$ alors que $f$ est négative dans un voisinage de $-\infty,$ ce qui constitue la contradiction recherchée!
  • Planche 21 : (Deuxième exo)

    C'est le genre d'exo qui fait appel à des arguments archi-classiques : le caractère multiplicatif des fonctions arithmétiques en jeu (une fois que l'on a vu le truc, on s'en souvient! :p)

    Notons pour $n\geq 1,$ $\displaystyle g(n)=\sum_{d|n}f(d).$
    La fonction $g$ est multiplicative i.e. si $a,b\geq 1$ et vérifie $a\wedge b=1$ alors $g(ab)=g(a)g(b)$ (ceci découle du lemme de Gauss).

    Soit $p\in \mathbb{P}.$
    Pour $\alpha\geq 1,$ il suffit alors de calculer $g(p^{\alpha})$ pour déterminer complètement $g$ sur $\mathbb{N}^{*}.$

    Or, par définition, on a :
    \begin{align*}
    g(p^{\alpha}) & =\sum_{k=0}^{\alpha}(-1)^{f(p^{k})}\\
    & =\sum_{k=0}^{\alpha}(-1)^{k}\\
    & =0 \mbox{ si } \alpha=1 [2] \mbox{ et } 1 \mbox{ si } \alpha=0 [2].
    \end{align*}

    Ainsi, $g(n)=0$ si l'une des valuations $p$-adiques de $n$ est impair et sinon $g(n)=1$ (si tous les valuations $p$-adiques de $n$ sont pairs).
  • Planche 19 (Exercice 3)

    -Pour $n=1,$ ce n'est pas possible (car $P(X)=X+2$ est de degré $1$).
    -Pour $n\geq 2,$ on suppose qu'une telle décomposition existe.
    On écrit $\displaystyle P(X)=\left(a_{k}X^{k}+\sum_{l=0}^{k-1}a_{l}X^{l}\right)\times \left(b_{k'}X^{k'}+\sum_{l=0}^{k'-1}b_{l}X^{l}\right)$ où $k,k'\geq 1$ avec $k+k'=n$ et les coefficients $a_{l},b_{l}\in\mathbb{N}.$

    On a clairement $a_{k}=1=b_{k'}.$

    En regardant le coefficient constant de $P,$ on a : $\displaystyle a_{0}b_{0}=p_{n}.$ On a ainsi, $a_{0}=1;b_{0}=p_{n}$ ou $a_{0}=p_{n};b_{0}=1.$ Quitte à renommer les coefficients, on peut supposer que $a_{0}=1;b_{0}=p_{n}.$

    -Notons $\displaystyle \min(k,k'):=u\geq 1.$

    Montrons par récurrence finie que : $a_{l}=0$ si $l\in\{1,\ldots,u-1\}$ et $b_{l}=p_{n-l}$ si $l\in\{0,\ldots,u-1\}.$

    *En regardant le coefficient de $X$ du polynôme $P,$ on a : $\displaystyle b_{1}+a_{1}p_{n}=a_{0}b_{1}+a_{1}b_{0}=p_{n-1}.$

    On a trois cas :

    -ou $k=1$ et alors $a_{1}=1$ alors que $p_{n}>p_{n-1},$ une contradiction!
    -ou $k'=1$ et alors nécessairement, $a_{1}=0$ mais $b_{1}=1,$ une contradiction! (car alors, on aurait $1=p_{n-1}$).
    -ou $k,k'\geq 2$ mais alors $a_{1}=0$ (car $p_{n}>p_{n-1}$) et ainsi $b_{1}=p_{n-1}.$

    Ceci achève l'initialisation.

    **Supposons que $a_{l}=0$ si $l\in\{1,\ldots,v\}$ et $b_{l}=p_{n-l}$ si $l\in\{0,\ldots,v\}$ où $v<u-1.$

    En regardant le coefficient de $X^{v+1}$ du polynôme $P,$ on a par hypothèse de récurrence : $\displaystyle b_{v+1}+a_{v+1}p_{n}=a_{0}b_{v+1}+a_{v+1}b_{0}=\sum_{l=0}^{v+1}a_{l}b_{v+1-l}=p_{n-v-1}.$

    On a trois cas :

    -ou $k=v+1$ et alors $a_{v+1}=1$ alors que $p_{n}>p_{n-v-1},$ une contradiction!
    -ou $k'=v+1$ et alors nécessairement, $a_{v+1}=0$ mais $b_{v+1}=1,$ une contradiction! (car alors, on aurait $1=p_{n-v-1}$).
    -ou $k,k'\geq v+1$ mais alors $a_{1}=0$ (car $p_{n}>p_{n-v-1}$) et ainsi $b_{1}=p_{n-v-1}.$

    Ceci achève l'hérédité. D'où le résultat annoncé!

    ***On applique le résultat prouvé par récurrence pour aboutir à la contradiction voulue!

    On a deux cas :

    -ou u=k (i.e. $k'\geq k$) En regardant le coefficient de $X^{k}$ du polynôme $P,$ on a alors grâce au résultat démontré précédemment : $\displaystyle b_{k}+p_{n}=a_{0}b_{k}+a_{k}b_{0}=\sum_{l=0}^{k}a_{l}b_{k-l}=p_{n-k}.$ Il s'agit d'une contradiction car $p_{n}>p_{n-k}$ (vu que $k\geq 1$).

    -ou u=k' (i.e. $k\geq k'$) En regardant le coefficient de $X^{k}$ du polynôme $P,$ on a alors grâce au résultat démontré précédemment : $\displaystyle 1+a_{k}p_{n}=a_{0}b_{k}+a_{k}b_{0}=\sum_{l=0}^{k}a_{l}b_{k-l}=p_{n-k}.$ On alors nécessairement $a_{k}=0$ (vu que $p_{n}>p_{n-k}$) et ainsi, on obtient $1=p_{n-k}$ qui est une contradiction!
  • edit Non utile
    Le 😄 Farceur


  • Ta preuve à l'air pas mal et plus intuitive que la mienne (je n'ai pas tout lu).

    Dans la preuve que j'annonçais le point crucial est d'éliminer les racines de module plus petit que $1$, et c'est pas évident : en écrivant $p_n=-\sum p_ix^{n-i}$ l'inégalité triangulaire est très mauvaise. Précisément parce on a la monotonie des coefficients, on voudrait faire apparaître des $p_n-p_{n-1}$ et pour cela on applique un truc qui est en fait bien connu sur le forum (cf. le lien dans le message plus haut), on regarde $(X-1)P=\sum (p_i-p_{i-1})X^{n-i}$ et l'inégalité triangulaire est une égalité (les coefficients du polynôme de droite somment à $p_n$) de sorte que la seule racine de module $\leq 1$ est $1$ qui n'est pas racine de $P$.
  • Exercice 26.1. Dans le cas surjectif et $E=F$ c'est le théorème de Mazur-Ulam dont la preuve est franchement difficile bien que parfaitement élémentaire. Le cas euclidien est un exercice de taupe : par polarisation on a conservation du produit scalaire et il s'en suit que en développant $\|f(x+y)-f(x)-f(y)\|^2$ comme un bourrin on obtient que cette norme est nulle, d'où le résultat.
  • Exercice 26 : Théorème bien connu (de géométrie des Banach) sous le nom de Théorème de Mazur-Ulam dont voici un lien: https://fr.wikipedia.org/wiki/Théorème_de_Mazur-Ulam

    La surjectivité est nécessaire (même si avec l'hypothèse que l'espace de Banach à l'arrivée a une norme strictement convexe, la conclusion du théorème reste vraie) pour pouvoir conclure au caractère affine (ou linéaire à normalisation près...) de l'isométrie.

    Montrons le théorème de plongement de Kuratwoski.

    Soit $(E,\|.\|)$ un espace de Banach séparable.
    Notons $(x_{n})_{n\in\mathbb{N}}$ une famille dense dans $E$ et prenons un point $y$ quelconque de $E.$
    Considérons alors l'application $\displaystyle \phi : E \rightarrow l^{\infty}(\mathbb{N})$ telle que pour $x\in E,$ $\displaystyle \phi(x)=(\|x-x_{n}\|-\|y-x_{n}\|)_{n\geq 0}.$

    L'application $\phi$ est alors une isométrie qui n'est manifestement ni affine et encore moins linéaire!
    Tout simplement car $\phi$ n'a aucune raison d'être surjective!
  • Puisque les solutions tombent
    Serait-il possible d'avoir presque ou toutes les solutions dans un fichier PDF ?
    Merci
  • Rédiger tout au propre dans un pdf prend beaucoup de temps, donc pas clair que ce soit fait. En tout cas ça n'est pas à l'ordre du jour tant que les autres exos ne seront pas tombés. Tu peux toujours imprimer la page dans un fichier pdf ou bien faire du copier coller pour mettre sur tex ...
  • *Planche 5 (Exercice 1... pour le plaisir de l'écrire car Mickaël l'a bien détaillé! ^^)

    Soit $f$ un quasi-morphisme multiplicatif (quasi-caractère?) de $G$ dans $\mathbb{C}.$

    -Ou il existe une constante $C>0$ telle que pour tout $x\in \mathbb{G},$ $\displaystyle \vert f(x) \vert \leq C.$

    -Ou il existe une suite d'éléments de $G$ : $\displaystyle (x_{n})\in G^{\mathbb{N}}$ telle que $\displaystyle \lim_{n\rightarrow +\infty} \vert f(x_{n})\vert =+\infty.$

    Soit $x,y\in G.$
    On note alors pour $n\gg 1,$ $a_{n}=\vert f(x_{n})\vert.$
    On a alors d'une part pour $n\gg1,$ $$B_{n}:=\left \vert \frac{f(xyx_{n})}{f(x_{n})}-f(xy) \right\vert =\frac{ \vert f(xyx_{n})-f(xy)f(x_{n}) \vert }{a_{n}}\leq \frac{\delta}{a_{n}}\longrightarrow_{n\rightarrow +\infty} 0.$$

    Et d'autre part, on a également pour $n\gg 1,$
    \begin{align*}
    C_{n}:=\left\vert \frac{f(xyx_{n})}{f(x_{n})}-f(x)f(y) \right\vert & \leq \frac{ \vert f(xyx_{n})-f(x)f(yx_{n})\vert }{a_{n}}+ \left\vert \frac{f(x)f(yx_{n})}{f(x_{n})}-f(x)f(y) \right\vert\\
    & \leq \frac{\delta}{a_{n}}+ \frac{\vert f(x) \vert }{a_{n}}\vert f(yx_{n})-f(y)f(x_{n})\vert\\
    & \leq \frac{\delta(1+\vert f(x) \vert )}{a_{n}}\longrightarrow_{n\rightarrow +\infty} 0.
    \end{align*}

    Ainsi, par l'inégalité triangulaire, on a $$\vert f(xy)-f(x)f(y)\vert \leq B_{n}+C_{n}\longrightarrow_{n\rightarrow +\infty}0.$$
    D'où le fait que $f$ est un morphisme multiplicatif (caractère) de $G$ i.e. $\displaystyle f(xy)=f(x)f(y).$

    **Planche 24 (Exercice 1... pour le plaisir d'achever les exos sur les quasi-morphismes! ^^)

    -Construisons $Q$ et montrons sa proximité avec $M$

    Soit $g\in G.$
    En notant pour $n\geq 0,$ $\displaystyle u_{n}=M(g^{2^{n}}) \mbox{ et } v_{n}=\frac{u_{n}}{2^{n}},$ on a pour une certaine constante $C>0$ $$\vert u_{n+1}-2u_{n}\vert \leq C \mbox{ et } \vert v_{n+1}-v_{n}\vert \leq \frac{C}{2^{n}}.$$
    La série de terme général $v_{n+1}-v_{n}$ converge absolument (dans le programme de prépa, on doit invoquer ce type d'argument car la notion de suite de Cauchy n'est pas au programme...) et donc $(v_{n})$ a une limite que l'on note $Q(g).$

    On remarque également $$\vert v_{N+1}-v_{0}\vert \leq \sum_{k=0}^{N}\vert v_{k+1}-v_{k}\vert \leq \sum_{k=0}^{N}\frac{C}{2^{k}}\leq 2C$$ et donc en passant à limite : $\displaystyle \vert Q(g)-M(g)\vert \leq 2C,$ d'où la propriété désirée de $Q$ "en passant l'inégalité au $\sup$ sur $G.$"

    Cette dernière propriété montre d'ailleurs que $Q$ est également un quasi-morphisme.
    Il suffit d'écrire : $\displaystyle \vert Q(g)+Q(h)-Q(gh)\vert \leq \vert M(g)+M(h)-M(gh)\vert +\vert Q(g)-M(g)\vert +\vert Q(h)-M(h)\vert +\vert Q(gh)-M(gh)\vert.$

    -Montrons que $Q$ est un quasi-caractère.

    Soit $g\in G.$ Notons $e$ le neutre de $G$.
    Soit $n\in \mathbb{N}$ et considérons $k\in \mathbb{Z}.$

    *ou $k\in \mathbb{N}.$
    On a alors pour tout $k\geq 1,$ $\displaystyle \vert M(g^{k})-kM(g) \vert \leq (k-1)C.$

    En effet, l'initialisation est claire et par l'inégalité triangulaire, on a : $$\vert M(g^{k+1})-(k+1)M(g) \vert \leq \vert M(g^{k+1})-M(g)-M(g^{k})\vert +\vert M(g^{k})-kM(g)\vert \leq kC.$$

    En appliquant l'inégalité précédente à $g$ remplacé par $g^{2^{n}}$, il vient alors pour $k\geq 1,$ $\displaystyle \vert \frac{M((g^{k})^{2^{n}})}{2^{n}}-k\frac{M(g^{2^{n}})}{2^{n}}\vert \leq \frac{(k-1)C}{2^{n}}.$
    Ainsi, en passant à la limite, il vient $\displaystyle Q(g^{k})=kQ(g).$

    Comme $\displaystyle M(e^{2^{n}})=M(e),$ il vient également $\displaystyle Q(g^{0})=Q(e)=0=0Q(g).$

    **ou $k\in \mathbb{Z}^{-*}.$

    En écrivant que $\displaystyle g^{k}=(g^{-1})^{-k},$ on obtient par le premier point $\displaystyle Q(g^{k})=-kQ(g^{-1}).$
    Cependant, comme pour tout $n\geq 0,$ $\displaystyle \vert M(g^{2^{n}})+M((g^{-1})^{2^{n}})-M(e)\vert \leq C,$ il vient en divisant par $2^{n}$ et en faisant tendre $n$ vers $+\infty,$ $\displaystyle Q(g)=-Q(g^{-1}).$
    Et finalement, $\displaystyle Q(g^{k})=kQ(g).$

    Ainsi, $Q$ est bien un quasi-caractère.

    -Montrons que $Q$ est l'unique quasi-caractère qui est à distance bornée de $M.$

    Soit $Q_{1}$ un autre quasi-caractère à distance bornée de $M.$
    Soit $g\in G.$
    En notant $\tilde{Q}:=Q-Q_{1},$ on a alors par l'inégalité triangulaire : $\displaystyle \vert \tilde{Q}(g)\vert \leq \vert Q(g)-M(g)\vert + \vert Q_{1}(g)-M(g)\vert \leq C$ (pour une certaine constante absolue $C>0$).
    Mais $\tilde{Q}$ est également un quasi-caractère et donc pour $n\gg1,$ $\displaystyle \vert \tilde{Q}(g) \vert = \frac{\vert \tilde{Q}(g^{n}) \vert }{n}\leq \frac{C}{n}\longrightarrow_{n\rightarrow +\infty} 0.$
    Ainsi, $\tilde{Q}=0$ et $Q=Q_{1}.$
  • Planche 21 (Exercice 1)

    *Je vais d'abord supposer que $X$ et $Y$ sont à valeurs dans $\mathbb{N}^{*}.$

    Il suffit de prouver le résultat pour des $x$ entiers car les variables aléatoires en jeu sont à valeurs dans $\mathbb{N}^{*}.$

    -On a clairement poour tout $n\gg1,$ $\displaystyle \mathbb{P}(X\geq n+1)\leq \mathbb{P}(X\geq n).$

    -Soit $\varepsilon>0$ tel que $\varepsilon\ll1.$
    Pour tout $n\gg1,$ on a alors $\displaystyle \mathbb{P}(X+Y\geq n+1)\leq 2(1+\varepsilon)\mathbb{P}(X\geq n+1) \mbox{ et } \mathbb{P}(X\geq n)\leq \varepsilon.$

    Ensuite, on a en conditionnant pour tout $n\gg1$ :
    \begin{align*}
    \mathbb{P}(X\geq n) & =\mathbb{P}(X\geq n;Y\in\{1,\ldots,n-1\})+\mathbb{P}(X\geq n;Y\geq n)\\
    \mbox{ et } \mathbb{P}(Y\geq n) & =\mathbb{P}(Y\geq n;X\in\{1,\ldots,n-1\})+\mathbb{P}(Y\geq n;X\geq n).
    \end{align*}

    Comme les $X$ et $Y$ sont i.i.d, il vient par somme des deux relations précédentes et pour tout $n\gg1$ :
    \begin{align*}
    2\mathbb{P}(X\geq n) & = \mathbb{P}\big([X\geq n;Y\in\{1,\ldots,n-1\}]\cup[Y\geq n;X\in\{1,\ldots,n-1\}] \big)+2\mathbb{P}^{2}(X\geq n)\\
    & \leq \mathbb{P}(X+Y\geq n+1)+2\mathbb{P}^{2}(X\geq n)\\
    & \mbox{ car l'union des évènements en jeu est incluse dans } [X+Y\geq n+1]\\
    &\leq 2(1+\varepsilon)\mathbb{P}(X\geq n+1) +2\varepsilon \mathbb{P}(X\geq n).
    \end{align*}

    Il vient alors pour tout $n\gg 1,$ $$\mathbb{P}(X\geq n)\leq \frac{1+\varepsilon}{1-\varepsilon}\mathbb{P}(X\geq n+1).$$

    Et ainsi, on a effectivement $\displaystyle \mathbb{P}(X\geq n)\sim_{n\rightarrow +\infty} \mathbb{P}(X\geq n+1).$

    **Il reste maintenant à traiter le cas où $X,Y$ sont à valeurs dans $\mathbb{N}.$

    Il s'agit juste d'observer dans la preuve précédente une "sorte de compensation" (bootstrap).

    -Soit $\varepsilon>0$ tel que $\varepsilon\ll1$ tel que $\displaystyle P(X=0):=c<1-\varepsilon$ (ce qui est possible si $X$ n'est pas supposée constante ce qui est un cas facile!).
    On note alors $\displaystyle l=1-c-\varepsilon>0.$

    Pour tout $n\gg1,$ on a alors $\displaystyle \mathbb{P}(X+Y\geq n+1)\leq 2(1+\varepsilon)\mathbb{P}(X\geq n+1) \mbox{ et } \mathbb{P}(X\geq n)\leq \varepsilon.$

    Ensuite, on a en conditionnant pour tout $n\gg1$ :
    \begin{align*}
    \mathbb{P}(X\geq n) & =\mathbb{P}([X\geq n;Y\in\{1,\ldots,n-1\}]\cup[X\geq n+1;Y=0])+\mathbb{P}(X=n;Y=0)+\mathbb{P}(X\geq n;Y\geq n)\\
    \mbox{ et } \mathbb{P}(Y\geq n) & =\mathbb{P}([Y\geq n;X\in\{1,\ldots,n-1\}]\cup[Y\geq n+1;X=0])+\mathbb{P}(Y=n;X=0)+\mathbb{P}(Y\geq n;X\geq n).
    \end{align*}

    Comme les $X$ et $Y$ sont i.i.d, il vient par somme des deux relations précédentes et pour tout $n\gg1$ :
    \begin{align*}
    2\mathbb{P}(X\geq n) & \leq \mathbb{P}(X+Y\geq n+1)+2c\mathbb{P}(X=n)+2\mathbb{P}^{2}(X\geq n)\\
    & \mbox{ car l'union des deux évènements incomptabiles }\\
    & \mbox{ apparaissant dans chacun des deux premiers termes est incluse dans } [X+Y\geq n+1]\\
    &\leq 2(1+\varepsilon)\mathbb{P}(X\geq n+1) +2\varepsilon \mathbb{P}(X\geq n)+2c\mathbb{P}(X=n).
    \end{align*}

    Il vient alors pour tout $n\gg 1,$ $\displaystyle (1-\varepsilon)\mathbb{P}(X\geq n)\leq (1+\varepsilon)\mathbb{P}(X\geq n+1)+c\mathbb{P}(X=n).$

    Comme $\displaystyle \mathbb{P}(X\geq n)-\mathbb{P}(X\geq n+1)=\mathbb{P}(X=n),$ on obtient ainsi pour tout $n\gg1,$ $\displaystyle (1-\varepsilon)\mathbb{P}(X=n)\leq 2\varepsilon\mathbb{P}(X\geq n+1)+c\mathbb{P}(X=n).$

    Il vient alors pour tout $n\gg 1,$ $\displaystyle \mathbb{P}(X=n)\leq \frac{2\varepsilon}{l}\mathbb{P}(X\geq n+1).$

    En écrivant $\displaystyle \mathbb{P}(X\geq n)=\mathbb{P}(X=n)+\mathbb{P}(X\geq n+1)\leq \left(1+\frac{2\varepsilon}{l}\right)\mathbb{P}(X\geq n+1),$ on a effectivement $\displaystyle \mathbb{P}(X\geq n)\sim_{n\rightarrow +\infty} \mathbb{P}(X\geq n+1).$
  • Bonjour,

    Pour l'exercice $1$ de la planche $36$, j'ai déterminé $\boxed{\:\text{la probabilité P qu'il existe} \:n \in \N^* \:\text {tel que }\:Y_n= 0.}$

    Je note: $\: \forall n \in \N,\:\: X_n =(Y_{n+2};Y_{n+3}),\:\:\quad E_n= \Big[X_n =(1,0)\Big] \bigcap \left[ \:\displaystyle \bigcap_{k=0}^{n-1} \:[X_k \neq (1,0)]\:\right ] ,\quad p_n = \Pr(E_n).\quad$ Alors:$$ P= \displaystyle \sum _{n=0} ^{+\infty} p_n.$$

    $\bullet \:G=(V,E)$ est le graphe orienté où:
    $\bullet \:V$ est l'ensemble des sommets défini par: $V=\Big\{(a,b) \in \N^2 \mid \exists n\in \N,\: \Pr \big[X_n =(a,b)\big] \neq 0 \Big\} \setminus \{(1;0)\}.$
    $ \bullet \;E$ est l'ensemble des arêtes défini par: $E=\Big \{ \big((a,b) ,(c,d) \big) \in V^2 \mid \ b=c\:\text{ et }\:d = | a \pm b | \Big \}. $
    $$\xymatrix @C-2pc{&amp;&&&&&&(1,1) \ar[rrrrd] ^{+}\\&&&(2,1) \ar[rrrru]^{-} \ar[rrd] ^{+}&&&&&&&& (1,2) \ar[llllllll] _{-}\ar[rrd] ^{+}\\&(3,2)\ar[rru] ^{-} \ar[rd]^{+}&&&&(1,3) \ar[llll] _{-} \ar[rd]^{+}&&&& (3,1) \ar[rru] ^{-} \ar[rd]^{+} &&&&(2,3) \ar[llll]_{-} \ar[rd]^{+}\\ (5,3) \ar[ru] ^{-} &&(2,5) \ar[ll] _{-} &&(4,1) \ar[ru] ^{-} &&(3,4) \ar[ll] _{-} &&(4,3) \ar[ru] ^{-} && (1,4) \ar[ll]_{-}&& (5,2) \ar[ru] ^{-} && (3,5) \ar[ll] _{-}\\}$$
    Soit $a_n$ le nombre de circuits de $G$ de longueur $3n$ allant de $(1,1)$ à $(1,1)$ et $b_n$ le nombre de ces circuits qui ne visitent $(1,1)$ qu'à leurs extrémités. $\quad a_0 =b_0 =a_1 = b_1=1.\ \quad\quad p_{3n} = \dfrac {a_n}{2^{3n+1}},\quad p_n=0 ,\:\text{si} \: n \not \equiv 0 \mod 3.$
    On dispose alors des relations de récurrence:
    $$\forall n \in \N^*,\:\: a_n = \displaystyle \sum_{k=1}^n a_{n-k}\:b_k\:,\quad\quad b_n = \sum _{k=0} ^{n-1} a_{k}\: a_{n-1-k}. $$
    La seconde relation résulte du fait que le graphe $G_1$, obtenu en retirant à $G$ les trois arêtes du circuit $"(1,1)\:(1,2)\:(2,1)\:(1,1)"$ et le sommet $(1,1)$, est constitué de deux composantes connexes isomorphes à $G$.
    En notant: $\:\forall x \:\text{tel que}\: |x|\leqslant \frac18,\quad A(x)=\displaystyle \sum_{n=0} ^{+\infty} a_n \:x^n , \quad B(x) =\sum _{n=0}^n b_n\:x^n$, les relations précédentes s'écrivent:
    $$2\Big(A(x) -1 \Big) =A(x) B(x) -1,\quad B(x) -1 =x\Big (A(x) \Big)^².$$
    d'où l'on déduit : $xA(x)^³ - A(x) +1 =0, \quad$ puis, avec $P= \dfrac { A (2^{-3})} 2, \quad P = \dfrac {\sqrt5 -1}2.$
    $$\boxed { \Pr \Big[ \displaystyle \bigcap_{n=1}^{+\infty}\: [Y_n \neq 0]\Big]= \dfrac{3-\sqrt5}2.}$$
  • Exercice 23 (je ne trouve pas cela très facile pour un éléve de classe préparatoire... :()

    -Soit $\alpha\in]0,1[.$
    Notons $\displaystyle Z_{\alpha}=\sum_{k\geq 0}\alpha^{k}X_{k}$ où les $(X_{k})$ sont une suite de Rademacher i.i.d.
    Ensuite, on note $\displaystyle F_{\alpha}$ la fonction de répartition de $Z_{\alpha}.$
    On note également $\mu_{\alpha}$ la loi de $Z_{\alpha}$ et on introduit $\displaystyle K_{\alpha}=\left\{ \sum_{k\geq 0}\alpha^{k}\varepsilon_{k}\mbox{ }|\mbox{ } \forall k\geq 0,\mbox{ } \varepsilon_{k}\in\{-1,1\} \right\}$ le support de $\mu_{\alpha}$, qui est un compact de $\displaystyle [-\frac{1}{1-\alpha},\frac{1}{1-\alpha}]$ (en procédant par extraction diagonale).

    *Montrons que tout élément de $K_{\alpha}$ s'écrit de manière unique si $0<\alpha<\frac{1}{2}.$
    (on peut montrer en calculant la transformée de Fourier de $\mu_{\frac{1}{2}}$ -i.e. la fonction caractéristique de $Z_{\alpha}$- que $\mu_{\frac{1}{2}}$ est la mesure de Lebesgue sur $[-2,2]$ et donc $K_{\frac{1}{2}}=[-2;2]$).

    Soit $t\in K_{\alpha}.$ Supposons qu'il existe deux suites $\varepsilon,\tilde{\varepsilon}\in\{-1,1\}^\mathbb{N}$ telles que $\varepsilon\neq \tilde{\varepsilon}$ vérifiant $\displaystyle \sum_{k\geq 0}\alpha^{k}\varepsilon_{k}=t=\sum_{k\geq 0}\alpha^{k}\tilde{\varepsilon_{k}}.$
    Notons $k_{0}=\min\left\{ k\geq 0\mbox{ }|\mbox{ } \varepsilon_{k}\neq \tilde{\varepsilon_{k}} \right\}.$ On a alors $\displaystyle \alpha^{k_{0}}\vert \varepsilon_{k_{0}}-\tilde{\varepsilon_{k_{0}}}\vert =\big\vert \sum_{k\geq k_{0}+1}\alpha^{k}\left(\varepsilon_{k}-\tilde{\varepsilon_{k}}\right)\big\vert.$
    Par l'inégalité triangulaire, il vient $\displaystyle 2\alpha^{k_{0}}\leq 2\sum_{k\geq k_{0}+1}\alpha^{k} \mbox{ i.e. } 2\alpha^{k_{0}}\leq \frac{2\alpha^{k_{0}+1}}{1-\alpha}.$
    On a ainsi $\displaystyle 1\leq \frac{\alpha}{1-\alpha}$ ce qui donne $\alpha\geq \frac{1}{2}.$ Ceci est impossible compte-tenu du choix de $\alpha.$


    **Continuité de $F_{\alpha}$ lorsque $\alpha\in]0,\alpha_{0}[$ où $0<\alpha_{0}<\frac{1}{2}$ est à déterminer!

    Remarque : Le point clé est d'obtenir la continuité des $F_{\alpha}$ lorsque $\alpha\in]0,l[$ pour un certain $0<l<1.$

    Soit $t\in\mathbb{R}.$

    ou $t\notin K_{\alpha}$ : vu que $K_{\alpha}$ est compact alors $\displaystyle d(t,K_{\alpha})>0$ et alors il existe $0<\varepsilon\ll1$ tel que $\displaystyle [t-\varepsilon,t+\varepsilon]\cap K_{\alpha}=\emptyset.$
    Ainsi, pour tout $\vert h\vert \leq \varepsilon,$ on a $\displaystyle F_{\alpha}(t+h)=F_{\alpha}(t)$ et donc $F_{\alpha}$ est continue en $t$ (car localement constante dans un voisinage de $t$).

    ou $t\in K_{\alpha}$ : Soit $l\gg1.$ On écrit alors $\displaystyle t=\sum_{k\geq 0}\alpha^{k}\varepsilon_{k}=\sum_{k=0}^{l}\alpha^{k}\varepsilon_{k}+\sum_{k\geq l+1}\alpha^{k}\varepsilon_{k}:=\tilde{t}+\sum_{k\geq l+1}\alpha^{k}\varepsilon_{k}$ où pour tout $k\geq 0,$ $\varepsilon_{k}\in\{-1,1\}.$

    Soit $\displaystyle \vert h \vert \leq \frac{\alpha^{l+1}}{1-\alpha}.$ On a alors :
    \begin{align*}
    \vert F_{\alpha}(t+h)-F_{\alpha}(t)\vert & \leq \mathbb{P}\left( \vert X_{\alpha} -t\vert \leq \frac{\alpha^{l+1}}{1-\alpha} \right)\\
    & \leq \mathbb{P}\left( \vert X_{\alpha}-\tilde{t}\vert \leq \frac{2\alpha^{l+1}}{1-\alpha} \right) \mbox{ par l'inégalité triangulaire, en majorant le reste } \vert t-\tilde{t}\vert\\
    & := \mathbb{P}(A_{l}).
    \end{align*}

    -Montrons que $\displaystyle A_{l}\subset \bigcap_{k=0}^{l}\{ X_{k}=\varepsilon_{k} \}$.

    Supposons que ce ne soit pas le cas. Il existe alors une réalisation $\omega\in A_{l}$ et un indice $k\in\{0,\ldots,l\}$ tel que $X_{k}(\omega)\neq \varepsilon_{k}(\omega).$ On note alors $k_{0}$ l'indice minimal vis à vis de cette dernière propriété.
    On obtient alors par l'inégalité triangulaire inversée, la chaîne suivante d'inégalités :
    $$2\alpha^{k_{0}}-\frac{2\alpha^{k_{0}+1}}{1-\alpha}\leq \vert X_{\alpha}(\omega)-\tilde{t}\vert \leq \frac{2\alpha^{l+1}}{1-\alpha}.$$
    Il vient alors en simplifiant $\displaystyle \alpha^{l+1-k_{0}}\geq 1-2\alpha.$
    Comme $k_{0}\in\{0,\ldots,l\},$ on a la minoration suivante : $\displaystyle \alpha^{l+1-k_{0}}\leq \alpha$ (vu que $\alpha\in]0,1[).$
    Mais alors, on obtient $\displaystyle 3\alpha-1\geq 0.$ Ceci est impossible, si on pose $\alpha_{0}:=\frac{1}{3}$ et si on choisit $\alpha\in]0,\alpha_{0}[.$

    -On obtient ainsi
    \begin{align*}
    \vert F_{\alpha}(t+h)-F_{\alpha}(t)\vert & \leq \mathbb{P}\left( \bigcap_{k=0}^{l-1}\{ X_{k}=\varepsilon_{k} \} \right)\\
    & \leq \frac{1}{2^{l}} \mbox{ par indépendance des va en jeu.}
    \end{align*}

    Ainsi, $F_{\alpha}$ est continue en $t$ (il est alors aisé à ce stade de vérifier la définition de la continuité de $F_{\alpha}$ au point $t$).

    Ainsi, $F_{\alpha}$ est continue sur $\mathbb{R}$ et donc $\mu_{\alpha}$ est sans atomes.

    ***Astuce de convolution

    On a écrit alors : $\displaystyle Z_{\alpha}=\sum_{k\geq 0}(\alpha^{2})^{k}X_{2k}+\alpha\sum_{k\geq 0}(\alpha^{2})^{k}X_{2k+1}=Z_{1,\alpha^{2}}+\alpha Z_{2,\alpha^{2}}$ où $Z_{1,\alpha^{2}}$ et $Z_{2,\alpha^{2}}$ sont des copies indépendantes et de même loi que $Z_{\alpha^{2}}.$

    Ainsi, $\mu_{\alpha}$ se présente comme la convolution de deux mesures qui sont sans atomes si $\alpha^{2}<\alpha_{0}$ (par le premier point de la preuve). Ainsi, $\mu_{\alpha}$ est également sans atomes si $\alpha^{2}<\alpha_{0}$ et donc $F_{\alpha}$ est continue sur $\mathbb{R}$ si $\alpha^{2}<\alpha_{0}.$

    Par une récurrence directe, on obtient alors que pour tout $\alpha\in]0,1[,$ $F_{\alpha}$ est continue sur $\mathbb{R}.$
  • Une rédaction pour l'exercice 27.1 comme promis (un peu à la vite fait, je suis sur téléphone) :
    Avec la fonction $F(x):=y(x)\exp (-\int_0^x a(t)dt)$ on a $F'(x)=a(x)y(x)(y(x-r(x)-y(x))$, on va donc regarder la position de $y(x-r(x))$ par rapport à $y(x)$.

    Pour cela on remarque que l'équation étant retardée donc les valeurs de $y$ pour $x<0$ déterminent entièrement celles pour $x>0$ et cela de manière unique et donc si on suppose $y(x)\ge 0$ pour tout $x <0$ (et on le fait jusqu'à nouvel ordre) alors il en est de même sur $\mathbb{R}$ tout entier. Avec l'équation on obtient alors $y' \ge 0$ sur $\mathbb{R}_+$ et donc $y$ est croissante et l'hypothèse de l'énoncé nous dit justement que $(x-r(x))>0$ pour des $x$ grands de sorte que, toujours sous l'hypothèse $y\leq 0$ sur $\mathbb{R}_-$ on a $F' \leq 0$ et donc comme $F \ge 0$ on obtient la convergence souhaitée.

    Pour le cas général on coupe $y=y^+-y_-$ sur $\mathbb{R}_-$ et on propage sur $\mathbb{R}$ en répétant les arguments précédents pour le prolongement de sorte que finalement $y^+$ et $y^-$ convergent, d'où le résultat.
Connectez-vous ou Inscrivez-vous pour répondre.
Success message!